LSAT and Law School Admissions Forum

Get expert LSAT preparation and law school admissions advice from PowerScore Test Preparation.

 Administrator
PowerScore Staff
  • PowerScore Staff
  • Posts: 8919
  • Joined: Feb 02, 2011
|
#40620
Complete Question Explanation
(The complete setup for this game can be found here: lsat/viewtopic.php?t=15461)

The correct answer choice is (C)

The question stem establishes that the 3-2-1 distribution is in effect, with one dish on the bottom shelf, two dishes on the middle shelf, and three dishes on the top shelf. From the third rule, we can infer that dish 6 must be stored on the middle shelf:

PT69_Game_#2_#10_diagram 1.png
From the second rule, dish 2 must then be stored on the top shelf. Dishes 1 and 4 rotate between the middle shelf and the top shelf as a result of the fourth rule. Dish 3—the random—is the last remaining dish, and it must then be stored on the top shelf:

PT69_Game_#2_#10_diagram 2.png
Accordingly, answer choice (C) must be true and is correct.

Get the most out of your LSAT Prep Plus subscription.

Analyze and track your performance with our Testing and Analytics Package.